Find the unkown tensions and masses in the situation below

  • #1
Venturi365
12
3
Thread moved from the technical forums to the schoolwork forums
TL;DR Summary: I don't know if my procedure is correct in this excercise

I've tried to solve this problem but I find my solution unintuitive and I think I might be wrong.

First of all, applying Newton's Laws I calculated the value for ##T_1## like this:

$$
\begin{align}
\sum F_{x} &=0\\
\sin(60) \cdot T_{1}+80\, \mathrm{N}\cdot \cos(60) &=0\\
T_{1}&=\frac{-80\cdot\cos(60)}{\sin(60)}\\
T_{1} &\approx -46,2\, \mathrm{N}
\end{align}
$$

Here's the first thing that looks odd to me, because intuitively ##T_{1}## should be ##80\,\mathrm{N}## too, but It may be just a wrong hypothesis.

Then I apply the same law to the node of the three cords:

$$
\begin{align}
\sum F_{y}&=0\\
T_{1}\cdot\cos(60)+80\,\mathrm{N}\cdot\sin(60)+T_{2}&=0\\
T_{2}&=46,2\,\mathrm{N}\cdot\cos(60)-80\,\mathrm{N}\cdot\sin(60)\\
T_{2}&\approx -46,2\,\mathrm{N}
\end{align}
$$

Which is the same force as ##T_{1}##. Is my method ok or am I wrong at some point?

je7yz.png
 
Physics news on Phys.org
  • #2
Welcome, @Venturi365 !
What is the wording of this problem?
 
  • #3
Lnewqban said:
Welcome, @Venturi365 !
What is the wording of this problem?
It asks you to find the values of ##T_{1}##, ##T_{2}## and ##m## supposing that the cables have no mass and that ##g=9,81\,\mathrm{\frac{m}{s^{2}}}##. The wording is just as I wrote it in the title.
 
  • #4
Could you do a free body diagram for the node of the three cords?
Consider that the node must have a balance of horizontal and vertical forces.
The magnitude of T1 does not need to be 80 Newtons (imagine the extreme case in which the right half of the rope becomes vertical and its tension is equal to mg).
CNX_UPhysics_06_01_StopLight.jpg
 
Last edited:
  • #5
Venturi365 said:
Which is the same force as T1.
The three forces must balance at the joint. Therefore the resultant of T1 and T2 must be along the line of 'T3'. With these angles, that means it lies on the bisector of the angle between them, implying they are of equal magnitude.

Btw, I find your use of signs quirky. A tension is generally taken to be a magnitude. (You could adopt a convention in which tensions have one sign and compressions the other.) Here, your equations have T1 and T2 negative but T3 positive.

I guess that came from starting with the vectors form, but then you should have measured all angles from the same reference direction, making some trig values negative.
 
  • #6
haruspex said:
The three forces must balance at the joint. Therefore the resultant of T1 and T2 must be along the line of 'T3'. With these angles, that means it lies on the bisector of the angle between them, implying they are of equal magnitude.

Btw, I find your use of signs quirky. A tension is generally taken to be a magnitude. (You could adopt a convention in which tensions have one sign and compressions the other.) Here, your equations have T1 and T2 negative but T3 positive.

I guess that came from starting with the vectors form, but then you should have measured all angles from the same reference direction, making some trig values negative.

Taking all that in consideration I get the same result:

First I find ##T_{1}## knowing that, at the joint, there's an equilibrium in the ##x## axis in which ##T_{2}## doesn't participate (now with the angles corrected):

$$
\begin{align}
\sum F_{x} & =0 \\
\cos \left(30^\circ\right) ·T_{1}+\cos \left(180^\circ+60^\circ\right) ·80\,\mathrm{N} & =0 \\
\frac{\sqrt{ 3 }}{2}·T_{1}-\frac{1}{2}80\,\mathrm{N} & =0 \\
T_{1} & =\frac{80}{\sqrt{ 3 }} \\
T_{1} & \approx 46,18\,\mathrm{N}
\end{align}
$$

Then, I sum all the tensions in the ##y## axis:

$$
\begin{align}
\sum F_{y} & =0 \\
T_{1}·\sin \left(30^\circ\right) +T_{2}+80\,\mathrm{N}·\sin \left(180^\circ+60^\circ\right) & =0 \\
\frac{80}{2\sqrt{ 3 }}+T_{2}-\frac{80\sqrt{ 3 }}{2 } & =0 \\
T_{2} & =\frac{80\sqrt{ 3 }}{2}-\frac{80}{2\sqrt{ 3 }} \\
T_{2} & \approx 46,18\,\mathrm{N}
\end{align}
$$

And there it is, the same solution, maybe I'm just right and I'm being a paranoid, idk.

I'll calculate the mass, just for fun:

$$
\begin{align}
w & =T_{2} \\
mg & =46,18 \\ \\

& \to \boxed{\; g =9,18\,\mathrm{\frac{m}{s^{2}}} \;} \\ \\

m & =\frac{46,18}{9,18} \\
m & \approx 5,03\,\mathrm{kg}
\end{align}
$$

Thx for the signs advice, I didn't see that.
 
  • #7
Venturi365 said:
And there it is, the same solution, maybe I'm just right and I'm being a paranoid
I explained in post #5 that with the angles specified it was inevitable the two tensions would be the same.
 
  • #8
haruspex said:
I explained in post #5 that with the angles specified it was inevitable the two tensions would be the same.

Oh, yeah, I'm so sorry. I've been studying for a long time and rn is 3 am in Spain so my brain didn't translate that correctly, lmao.

Thank you for your patience and taking the time to answer :)
 
  • #9
Venturi365 said:
Oh, yeah, I'm so sorry. I've been studying for a long time and rn is 3 am in Spain so my brain didn't translate that correctly, lmao.

Thank you for your patience and taking the time to answer :)
No problem, you are welcome.
 

Related to Find the unkown tensions and masses in the situation below

What is the first step in finding unknown tensions and masses in a given system?

The first step is to draw a free-body diagram for each object in the system. This involves identifying all the forces acting on each object, including gravitational forces, tension forces, and any other applicable forces. Label these forces clearly on the diagram.

How do you apply Newton's laws to solve for unknown tensions and masses?

After drawing the free-body diagrams, apply Newton's second law (F = ma) to each object. Set up equations based on the sum of forces in the horizontal and vertical directions. For objects in equilibrium, the sum of forces will be zero. For objects in motion, the sum of forces will equal the mass times the acceleration.

What role do the angles of the ropes or strings play in determining tensions?

The angles of the ropes or strings are crucial because they affect the components of the tension forces. Use trigonometric functions (sine and cosine) to resolve the tension forces into their horizontal and vertical components. This will allow you to set up equations based on these components.

How can you solve for multiple unknowns in the system?

To solve for multiple unknowns, you will generally need as many independent equations as there are unknowns. Use the equations derived from applying Newton's laws to each object. Solve these simultaneous equations using algebraic methods such as substitution or matrix operations.

What are common mistakes to avoid when solving for unknown tensions and masses?

Common mistakes include neglecting to account for all forces acting on each object, incorrectly resolving forces into components, and making arithmetic errors when solving equations. Always double-check your free-body diagrams and calculations, and ensure that the units are consistent throughout the problem.

Similar threads

  • Introductory Physics Homework Help
Replies
7
Views
574
  • Introductory Physics Homework Help
Replies
5
Views
683
  • Introductory Physics Homework Help
Replies
6
Views
443
  • Introductory Physics Homework Help
Replies
8
Views
808
  • Introductory Physics Homework Help
Replies
6
Views
985
  • Introductory Physics Homework Help
Replies
15
Views
468
  • Introductory Physics Homework Help
Replies
5
Views
1K
  • Introductory Physics Homework Help
Replies
13
Views
1K
  • Introductory Physics Homework Help
Replies
3
Views
906
Replies
6
Views
2K
Back
Top